LSAT and Law School Admissions Forum

Get expert LSAT preparation and law school admissions advice from PowerScore Test Preparation.

 Administrator
PowerScore Staff
  • PowerScore Staff
  • Posts: 8919
  • Joined: Feb 02, 2011
|
#38947
Complete Question Explanation
(The complete setup for this game can be found here: lsat/viewtopic.php?t=15134)

The correct answer choice is (E)

The local condition requiring V to perform fourth can be satisfied in both templates, and so we need to create two local diagrams to represent the range of possible solutions. Given the specific variable placement required by the stem, the use of Basic Linear diagraming would be preferable to modifying our sequencing chains.

If V is fourth in Template 1, then K and G will be forced into positions five and six, respectively. The remaining variables—P, S, and T—will occupy the first three slots:
PT72 - Game_#1_#5_diagram 1.png
Placing V in the fourth position in Template 2 would have a similarly restrictive effect on all other variables:
PT72 - Game_#1_#5_diagram 2.png
With these two solutions in place, identifying which answer choice does not need to be true should be relatively simple.

Answer choice (A): This answer choice is incorrect, because P must perform before V in both solutions.

Answer choice (B): This answer choice is incorrect, because T must perform before V in both solutions.

Answer choice (C): This answer choice is incorrect, because T must perform before G in both solutions.

Answer choice (D): This answer choice is incorrect, because S must perform before V in both solutions.

Answer choice (E): This is the correct answer choice, because T need not perform S. The inverse is possible in Template 1 (above).

Get the most out of your LSAT Prep Plus subscription.

Analyze and track your performance with our Testing and Analytics Package.